Answer:
2,3,5,then the last one
Step-by-step explanation:
Answer:
1.The third one
2. The last one
3. The second one
4. The fifth one
Step-by-step explanation:
if Rachel moves 5 laws in 8 hours at this rate how many lawns can she mow in 40 hours?
Find m of MLJ
See photo below
Answer:
45°---------------------
The angle formed by a tangent and secant is half the difference of the intercepted arcs:
12x - 3 = (175 - 21x - 1)/224x - 6 = 174 - 21x24x + 21x = 174 + 645x = 180x = 4Find the measure of ∠MLJ by substituting 4 for x in the angle measure:
m∠MLJ = 12*4 - 3 = 48 - 3 = 45Recently, More Money 4U offered an annuity that pays 6.9% compounded monthly. If $2,185 is deposited into this annuity every month, how much is in the account after 5 years? How much of this is interest? Type the amount in the account: $ (Round to the nearest dollar.) Type the amount of interest earned: \$ (Round to the nearest dollar.)
After 5 years, the account will have approximately $158,523. The total interest earned over this period is approximately $37,523.
The calculation of the final amount in the account after 5 years involves compounding the monthly deposits with the given interest rate. To determine the total amount, we can use the formula for the future value of an annuity:
A = P * [(1 + r)^n - 1] / r,
where A is the future value, P is the monthly deposit, r is the monthly interest rate, and n is the number of periods (in this case, 5 years multiplied by 12 months per year).
Plugging in the values, we have:
P = $2,185
r = 6.9% / 100% / 12 = 0.00575 (monthly interest rate)
n = 5 * 12 = 60 (number of periods)
A = $2,185 * [(1 + 0.00575)^60 - 1] / 0.00575 ≈ $158,523.
To calculate the interest earned, we subtract the total deposits made over 5 years (60 months * $2,185) from the final amount:
Interest = $158,523 - (60 * $2,185) ≈ $37,523.
Therefore, after 5 years, the account will have approximately $158,523, with approximately $37,523 being the interest earned.
Learn more about interest here: https://brainly.com/question/8100492
#SPJ11
This topic is all about inequalities!
The question is in this image.
Considering the perimeter of the trapezoid, the set-builder representation of the possible values for the length of the sides labeled m is:
{m | m > 6}.
What is the perimeter of a polygon?The perimeter of a polygon is calculated by the sum of the lengths of all the outer edges of the figure.
In the context of this problem, these measures of the outer edges are given as follows:
6.10.m.m.Hence the perimeter of the trapezoid is given as follows:
P = 6 + 10 + m + m = 16 + 2m.
The perimeter is greater than 28, hence the inequality is:
P > 28.
16 + 2m > 28. (replacing the equation for P into the inequality).
2m > 12
m > 12/2
m > 6.
The set-builder notation of this interval is:
{m | m > 6}.
More can be learned about the perimeter of a figure at https://brainly.com/question/397857
#SPJ1
Hailey is the oldest of four siblings whose ages are consecutive integers. If the sum of their ages is 66, find Hailey's age.
Considering the definition of an equation and the way to solve it, Hailey's age is 18 years.
Definition of equationAn equation is the equality existing between two algebraic expressions connected through the equals sign in which one or more unknown values, called unknowns, appear in addition to certain known data.
The members of an equation are each of the expressions that appear on both sides of the equal sign while the terms of an equation are the addends that form the members of an equation.
The solution of a equation means determining the value that satisfies it. In this way, by changing the unknown to the solution, the equality must be true.
To solve an equation, keep in mind:
When a value that is adding, when passing to the other member of the equation, it will subtract.If a value you are subtracting goes to the other side of the equation by adding.When a value you are dividing goes to another side of the equation, it will multiply whatever is on the other side.If a value is multiplying it passes to the other side of the equation, it will pass by dividing everything on the other side.Hailey's ageBeing "x" the age of the youngest child, then you have:
"x" the age of the youngest child."x + 1" the age of one middle child."x + 2" the age of the other middle child."x + 3" the age of the oldest child, in this case Hailey's age.On the other hand, you know that the sum of their ages is 66. Then, the equation is:
x + x + 1 + x + 2 + x + 3 = 66
Solving:
4x + 6 = 66
4x = 66 - 6
4x = 60
x= 60 ÷ 4
x= 15
Remembering that "x + 3" is Hailey's age, then her age is calculated as: x+3= 15 +3= 18
Finally, Hailey's age is 18 years.
Learn more about equations:
brainly.com/question/4983716
brainly.com/question/13314678
#SPJ1
a car travels at a constant speed so that every minute it travels 1.4 km. Is this a proportional relationship?
If you increase the time taken, the distance traveled will also increase in a proportional manner.
Yes, this is a proportional relationship because the distance traveled is directly proportional to the time taken. The constant speed of the car means that for every minute, it travels the same distance of 1.4 km. Therefore, if you increase the time taken, the distance traveled will also increase in a proportional manner.
To know more about speed,distance and time, visit:
https://brainly.com/question/31756299
#SPJ11
help please please please please please please please please
Question 1= $7,620
Question 2 (a) $3,211
Question 2 (b) $9,828
Find the quarter point of y=1/2sin(2x+π)
Answer:
32
Step-by-step explanation:
y = 1/2sin[2(x + π/2)]. Now you can read that the graph is shifted from its standard starting position at (0, 0) left by π/2 and the period ...
-8÷2+12x9-4x6
56 + 7x2
\(\huge\textbf{Hey there!}\)
\(\huge\textsf{Question \#1.}\)
\(\mathbf{-8\div2 + 12 \times9 - 4\times6}\\\mathbf{= -4 + 12\times9 - 4\times6}\\\mathbf{= -4 + 108 - 4\times6}\\\mathbf{= 104 - 4\times6}\\\mathbf{= 104 - 24}\\\mathbf{= 80}\)
\(\huge\textsf{Therefore, your answer is: \boxed{\mathsf{80}}}\huge\checkmark\)
\(\huge\textsf{Question \#2.}\)
\(\mathbf{56 + 7\times2}\\\mathbf{= 56 + 14}\\\mathbf{= 70}\)
\(\huge\text{Therefore, your answer is: \boxed{\mathsf{70}}}\huge\checkmark\)
\(\huge\textbf{Good luck on your assignment \& enjoy}\\\huge\textbf{your day!}\)
~\(\frak{Amphitrite1040:)}\)Explanation of how we can make (a) subject
Answer:
Step-by-step explanation:
Simplify the polynomial (7m+5)^2
Answer:
\(49 {m}^{2} + 70m + 25 \)Solution,
( 7m + 5)^2
Using the formula:
\( {(a + b)}^{2} = {a}^{2} + 2ab + {b}^{2} \)
and expand the expression
= ( 7m)^2 + 2 • 7m • 5 + (5)^2
Evaluate the power
= 49m^2 + 2 • 7m • 5 + 25
Calculate the product
= 49m^2 + 70m + 25
Hope this helps...
Good luck on your assignment..
Helppppppppppppppppppppppppppppp!!!!!!!!!
Answer:
1. Yes
2.i don't know sorry
3. no
4. no
Step-by-step explanation:
1. it is a terminating decimal
2. I don't know sorry
3.it is a repeating decimal
4.itis a repeating decimal
Find the terms through degree 4 of the Maclaurin series of f. Use multiplication and substitution as necessary. f(x) = (1+x)^-4/3 (Express numbers in exact form. Use symbolic notation and fractions where needed.) f(x)=
The terms through degree 4 of the Maclaurin series of f(x) = (1+x)^-4/3 are 1, -4x/3, 16x^2/9, -64x^3/27, and 256x^4/81. This can be found using the Maclaurin series for (1+x)^-n, which is 1 - nx + n(n-1)x^2/2! - ... + (-1)^n n! x^n/n!.
The Maclaurin series for (1+x)^-n is 1 - nx + n(n-1)x^2/2! - ... + (-1)^n n! x^n/n!. This can be found using the Binomial Theorem. For example, the term -4x/3 comes from the coefficient of x in the expansion of (1+x)^-4/3, which is (-4/3)(-4/2) = 16/3. To find the terms through degree 4, we need to evaluate the first four terms of the Maclaurin series for (1+x)^-n. These terms are 1, -4x/3, 16x^2/9, and -64x^3/27.
Therefore, the terms through degree 4 of the Maclaurin series of f(x) = (1+x)^-4/3 are 1, -4x/3, 16x^2/9, -64x^3/27, and 256x^4/81.
To learn more about Maclaurin series click here : brainly.com/question/31745715
#SPJ11
The table shows the high temperature on the moon during the day and the overnight low temp.
time of day | temperature
day | 253
night | -387
what is the range between the moons minimum temp in degrees fahrenheit?
The range between the moon's minimum and maximum temperature in degrees Fahrenheit is 640°F
From the question, we have
Time of Day Temperature
Day 253°F
Night -387°F
the moon's minimum temperature = -387°F
and maximum temperature = 253°F
range = 253 - (-387) = 640°F
Subtraction:
Subtraction represents the operation of removing objects from a collection. The minus sign signifies subtraction −. For example, there are nine oranges arranged as a stack (as shown in the above figure), out of which four oranges are transferred to a basket, then there will be 9 – 4 oranges left in the stack, i.e. five oranges. Therefore, the difference between 9 and 4 is 5, i.e., 9 − 4 = 5. Subtraction is not only applied to natural numbers but also can be incorporated for different types of numbers.
The letter "-" stands for subtraction. Minuend, subtrahend, and difference are the three numerical components that make up the subtraction operation. A minuend is the first number in a subtraction process and is the number from which we subtract another integer in a subtraction phrase.
Complete question:
The table shows the high temperature on the moon during the day and the overnight low temperature.
Time of Day Temperature
Day 253°F
Night –387°F
What is the range between the moon's minimum and maximum temperature in degrees Fahrenheit?
To learn more about subtraction visit: https://brainly.com/question/2346316
#SPJ4
If you are good at 9th grade math tell me and send me your number so we can text I really need a tutor or just help please
Answer:
I'm super sorry that i can't help but if i could get you to my brother and ask him for his number i bet he'll be able to help. ;) and if u r a girl he's pretty desperate for a girlfriend so ..... yeah
Step-by-step explanation:
hope this helps and also hope my brother can help byyyyyeeeee
Solve for f(5):
f(5) = -x + 4
Answer: look at the picture
Step-by-step explanation: hope this help
PLEASE PLEASE PLEASEEE! THIS IS DUE TOMORROW! ILL THANKS AND WHAT EVER ELSE I CAN TO WHOEVER GETS THIS!
1.Y=-2/6 -5
2. >
3.<
4. False because the y intercept could be a positive or negative.
That's the best I got.
Let U be a square matrix (a general matrix), such that UTU=I. Show that detU=±1.
Using the fact that det(I) = 1, this simplifies to: det(U)^2 = 1. Therefore, we have shown that det(U) must be either 1 or -1.
To show that det(U) = ±1 for a square matrix U, such that U^T * U = I, follow these steps:
1. We know that U is a square matrix, so it has the same number of rows and columns.
2. The given condition is U^T * U = I, where U^T is the transpose of U, and I is the identity matrix.
3. Calculate the determinant of both sides of the equation. Using the property det(AB) = det(A) * det(B), we have:
det(U^T * U) = det(I)
4. The determinant of the identity matrix I is always 1, so:
det(U^T * U) = 1
5. Using the property det(U^T) = det(U), we can rewrite the left side of the equation as:
det(U^T) * det(U) = det(U) * det(U) = det(U)^2
6. So we have:
det(U)^2 = 1
7. Taking the square root of both sides, we get:
det(U) = ±1
Thus, for a general square matrix U satisfying the given condition, its determinant is either 1 or -1.
to know more about square matrix click here:
https://brainly.com/question/13179750
#SPJ11
Two particles have positions at time t given by s1=4t-t^2 and s2=5t^2-t^3. Find the velocities
Step-by-step explanation:
Given that,
The positions of the first particle,\(s_1=4t-t^2\)
Velocity,
\(v_1=\dfrac{ds_1}{dt}\\\\v_1=\dfrac{d(4t-t^2)}{dt}\\\\v_1=4-2t\)
Position,
\(s_2=5t^2-t^3\)
\(v_2=\dfrac{ds_2}{dt}\\\\v_2=\dfrac{d(5t^2-t^3)}{dt}\\\\v_2=10t-3t^2\)
Hence, this is the required solution.
mrs. alvarez rents skis and poles for 3 days what is the total cost of the rental
skis per day is a total of 180 for three days
Poles per day is 36 which totals 180
Subtract the 18 dollars for the coupon
162 is the answer
Step-by-step explanation:
Everythings in the picture!
Answer:
See below
Step-by-step explanation:
Year Starting balance,$ prt, $ Interest,$
1 1400 1400(1+0.06) = 1484.00 84
2 1484 1484(1+0.06) = 1573.04 89.04
3 1573.04 1573.04(1+0.06) = 1667.42 94.38
The table shows values for a quadratic function.
x,y
0,0
1,2
2,8
3,18
4,32
5,50
6,72
What is the average rate of change for this function for
the interval from x= 1 to x= 3?
A. 6
B. 4
C. 8
D. 9
The a = 0.Substituting the values of a, b, and c in the general equation, we get:y = 0x² + 2x + 3The quadratic function is:y = 2x + 3Answer: The quadratic function is y = 2x + 3.
The given table illustrates the values of a quadratic function. Here is how you can find the quadratic function:Step 1: Write the general form of a quadratic function y = ax² + bx + c, where y is the dependent variable and x is the independent variable. a, b, and c are constants that affect the shape and position of the parabola.Step 2: Substitute the values from the table for x and y to form a system of equations.Step 3: Solve the system of equations to find the values of a, b, and c. Once you have found these values, substitute them into the quadratic equation to get the quadratic function.
The given table is as follows:x | 0 | 2 | 4 | 6y | 3 | 1 | -1 | -3Step 2:Form a system of equations using the values in the table. Here are the equations:y = a(0)² + b(0) + cy = a(2)² + b(2) + cy = a(4)² + b(4) + cy = a(6)² + b(6) + cStep 3:Solve the system of equations.Using the first equation, y = c. Hence, we have:y = 0²a + 0b + c3 = cThe value of c is 3.Using the second equation, we have:y = 2²a + 2b + 3y = 4a + 2b + 3Subtracting the two equations, we get:- 2a - b = - 2a + b = 2b = 4Therefore, b = 2.Substituting the values of b and c into the first equation, we get:3 = a(0)² + 2(0) + 3
for more search question equation
https://brainly.com/question/29174899
#SPJ8
(1). Consider the 3×3 matrix 1 1 1 2 1 003 A = 0 Find the sum of its eigenvalues. a) 7 b) 4 c) -1 d) 6 e) none of these
The sum of eigenvalues of a matrix A is equal to the trace of matrix A. Here, the trace is 5, so the sum of eigenvalues is 5.
Trace of a square matrix is the sum of its diagonal entries. Eigenvalues of a square matrix are the values which satisfy the equation det(A- λI) = 0, where I is the identity matrix of the same size as A. Here, the given matrix A is a 3x3 matrix with its diagonal entries as 1, 1, and 3.
Therefore, trace(A) = 1+1+3 = 5.
Also, det(A- λI)
= (1- λ) [ (1- λ)(3- λ) - 0] - (1) [ (2)(3- λ) - 0] + (1) [ (2)(0) - (1)(1- λ)]
= λ3 - 5λ2 + 6λ - 2
= (λ - 2)(λ - 1)(λ - 1).
Now, the eigenvalues are 2, 1 and 1. The sum of these eigenvalues is 2+1+1 = 4.
Therefore, option (b) 4 is incorrect. The correct answer is option (a) 7 as the sum of the eigenvalues of matrix A is equal to the trace of matrix A which is 5.
Learn more about identity matrix here:
https://brainly.com/question/2361951
#SPJ11
400,000,008,912 – 88,888,116,803 = PLEASE HELP MEEEEEEEEEEEE
As a result of answering the given question, we may state that Therefore, expressions the answer to your subtraction problem is 311,111,892,109.
what is expression ?An expression in mathematics is a combination of numbers, variables, and mathematical operations (such as addition, subtraction, multiplication, division, exponentiation, and so on) that expresses a quantity or a value. Expressions might be simple, such as "3 + 4", or complex, such as "(3x2 - 2) / (x + 1)". They can also include functions like "sin(x)" or "log(y)". Expressions can be evaluated by substituting values for the variables and performing the mathematical operations in the order specified. For example, if x = 2, the formula "3x + 5" evaluates to 3(2) + 5 = 11. Expressions are frequently used in mathematics to express real-world situations, create equations, and simplify complex mathematical problems.
When you subtract 88,888,116,803 from 400,000,008,912, you get:
400,000,008,912 - 88,888,116,803 = 311,111,892,109
Therefore, the answer to your subtraction problem is 311,111,892,109.
To know more about expressions visit :-
https://brainly.com/question/14083225
#SPJ1
Read each sentence. Then, use the drop-down menus to identify the types of figurative language used. I must be popular because my phone is blowing up. Her smile lit up my day. The dog continued to bark, speaking with authority. He wept like a faucet.
Answer:
I must be popular because my phone is blowing up.
✔ hyperbole
Her smile lit up my day.
✔ metaphor
The dog continued to bark, speaking with authority.
✔ personification
He wept like a faucet.
✔ simile
Step-by-step explanation:
Answer:
I must be popular because my phone is blowing up. hyperbole
Her smile lit up my day. metaphor
The dog continued to bark, speaking with authority. personification
He wept like a faucet. simile
Step-by-step explanation:
Perform a Radix sort on the sequence of numbers given as follows: 13,10,7,124,90,321,809,109,11,1 B. Use the heapify method to draw a maximum heap tree for the sequence: 15,5,20,1,17,10,30,16
The sorted sequence using Radix Sort is: 1, 7, 10, 11, 90, 109, 124, 321, 809. The root of the heap tree is 15.
Performing Radix Sort on the sequence: 13, 10, 7, 124, 90, 321, 809, 109, 11, 1:
Start with the least significant digit (rightmost digit):
1s place: 13, 10, 90, 321, 11, 1, 124, 7, 109, 809
Group the numbers based on their 1s place digit:
0: 10
1: 11, 1
2: 321
4: 124
7: 7
9: 90, 109, 809
(empty slots for digits 3, 5, 6, 8)
Concatenate the groups in order: 10, 11, 1, 321, 124, 7, 90, 109, 809
Repeat the process for the next significant digit:
10s place: 10, 11, 1, 7, 90, 109, 321, 809, 124
Group the numbers based on their 10s place digit:
0: 10, 90
1: 11, 1
2: 321
7: 7
8: 809
9: 109, 124
(empty slots for digits 3, 4, 5, 6)
Concatenate the groups in order: 10, 90, 11, 1, 321, 7, 809, 109, 124
Repeat the process for the next significant digit:
100s place: 1, 7, 10, 11, 90, 109, 124, 321, 809
Group the numbers based on their 100s place digit:
0: 1, 7, 10, 11, 90
1: 109
2: 124
3: 321
8: 809
(empty slots for digits 4, 5, 6, 7, 9)
Concatenate the groups in order: 1, 7, 10, 11, 90, 109, 124, 321, 809.
The following is the code to draw a maximum heap tree for the sequence:
def heapify(heap, index):
"""Heapifies the heap starting at the given index."""
largest = index
left = 2 * index + 1
right = 2 * index + 2
if left < len(heap) and heap[left] > heap[largest]:
largest = left
if right < len(heap) and heap[right] > heap[largest]:
largest = right
if largest != index:
heap[index], heap[largest] = heap[largest], heap[index]
heapify(heap, largest)
if __name__ == "__main__":
heap = [15, 5, 20, 1, 17, 10, 30, 16]
for i in range(len(heap) // 2, -1, -1):
heapify(heap, i)
print(heap)
The following is the output of the code:
[1, 5, 10, 16, 17, 20, 30, 15]
The root of the tree is 15, which is the largest element in the sequence. The children of the root are 5 and 20, and the grandchildren of the root are 1 and 16. The tree is a max heap because the value of each node is greater than or equal to the values of its children.
To learn more about Radix Sort here:
https://brainly.com/question/13326818
#SPJ4
Suriland cannot both export wheat and keep bread plentiful and affordable in Suriland. Accordingly, Suriland's wheat farmers are required to sell their crop to the government, which pays them a dollar per bushel less than the price on the world market. Therefore, if the farmers could sell their wheat on the world market, they would make a dollar per bushel more, less any additional transportation and brokerage costs they would have to pay.
Which of the following, if true, most seriously weakens the argument?
(A) Suriland's wheat farmers have higher production costs than do farmers in many other wheat-producing countries.
(B) Sale of a substantial proportion of Suriland's wheat crop on the world market would probably depress the price of wheat.
(C) The transportation and brokerage costs that Suriland's farmers would face if they sold their wheat outside Suriland could amount to almost a dollar per bushel.
(D) Suriland is surrounded by countries that do not import any wheat.
(E) The price of a bushel of wheat on the world market occasionally drops below the average cost of producing a bushel of wheat in Suriland.
Answer:
the most accurate argument is (C) The transportation and brokerage costs that Suriland's farmers would face if they sold their wheat outside Suriland could amount to almost a dollar per bushel.
Step-by-step explanation:
The cost of transportation and other logistics is being covered by the government that is why the government pay less than dollar per bushel.
so the price of the wheat would have to decrease at home so that the cost of the transportation can be covered by the government who are buying directly from the farmers.
use technology or a z-distribution table to find the indicated area.the scores on a standardized exam are normally distributed with a mean of 400 and a standard deviation of 50.approximately 40% of the scores are greater than which score?
Approximately 40% of the scores on the standardized exam are greater than a score of 412.5
To find the score for which approximately 40% of the scores are greater, using the z-distribution table, follow these steps:
1. Identify the given values: mean (µ) = 400, standard deviation (σ) = 50, and the area to the right of the z-score (40% or 0.40).
2. Find the corresponding area to the left (1 - 0.40 = 0.60 or 60%).
3. Look up the z-score in the z-distribution table that corresponds to an area of 0.60 to the left. You will find a z-score of approximately 0.25.
4. Use the z-score formula to find the corresponding score (x):
x = µ + (z × σ) = 400 + (0.25 × 50) = 400 + 12.5 = 412.5
Therefore, approximately 40% of the scores on the standardized exam are greater than a score of 412.5.
To know more about z-distribution table refer here :
https://brainly.com/question/28951428#
#SPJ11
The average weight of a chicken egg is 2.25 ounces with a standard deviation of 0.2 ounces. You take a random sample of a dozen eggs.
a) What are the mean and standard deviation of the sampling distribution of sample size 12?
b) What is the probability that the mean weight of the eggs in the sample will be less than 2.2 ounces?
The mean of the sampling distribution = 2.25 ounces and the standard deviation ≈ 0.0577 ounces and the probability that the mean weight of the eggs in the sample will be less than 2.2 ounces ≈ 0.1915 or 19.15%.
a) To calculate the mean and standard deviation of the sampling distribution of sample size 12, we can use the properties of sampling distributions.
The mean (μ) of the sampling distribution is equal to the mean of the population.
In this case, the average weight of a chicken egg is prvoided as 2.25 ounces, so the mean of the sampling distribution is also 2.25 ounces.
The standard deviation (σ) of the sampling distribution is equal to the population standard deviation divided by the square root of the sample size.
Provided that the standard deviation of the eggs' weight is 0.2 ounces and the sample size is 12, we can calculate the standard deviation of the sampling distribution as follows:
σ = population standard deviation / √(sample size)
= 0.2 / √12
≈ 0.0577 ounces
Therefore, the mean = 2.25 ounces, and the standard deviation ≈ 0.0577 ounces.
b) To calculate the probability that the mean weight of the eggs in the sample will be less than 2.2 ounces, we can use the properties of the sampling distribution and the Z-score.
The Z-score measures the number of standard deviations a provided value is away from the mean.
We can calculate the Z-score for 2.2 ounces using the formula:
Z = (x - μ) / (σ / √n)
Where:
x = value we want to obtain the probability for (2.2 ounces)
μ = mean of the sampling distribution (2.25 ounces)
σ = standard deviation of the sampling distribution (0.0577 ounces)
n = sample size (12)
Plugging in the values, we have:
Z = (2.2 - 2.25) / (0.0577 / √12)
≈ -0.8685
The probability that the mean weight of the eggs in the sample will be less than 2.2 ounces is the area under the standard normal curve to the left of the Z-score.
Using the Z-table or a calculator, we obtain that the probability is approximately 0.1915.
To know more about probability refer here:
https://brainly.com/question/32696302#
#SPJ11
The formula to convert °F to °C is C = C equals StartFraction 5 Over 9 EndFraction left-parenthesis F minus 32 right-parenthesis.(F – 32).
Convert 50°C to °F.
Answer: 122°F
Step-by-step explanation:substitute 50°C anywhere in the formula that has the C